Sei sulla pagina 1di 90

Ftplectures Cardiovascular system Lecture Notes

CARDIOLOGY

Medicine made simple

This content is for the sole use of the intended recipient(s) and may contain information that is proprietary,
confidential, and exempt from disclosure under applicable law. Any unauthorized review, use, disclosure, or
distribution is prohibited. All content belongs to FTPLECTURES, LLC. Reproduction is strictly prohibited.

COPYRIGHT RESERVED

Ftplectures Cardiovascular system
Copyright 2014

Adeleke Adesina, DO
Cardiovascular system

2012 ftplectures LLC


1133 Broadway Suite 706,
New York, NY, 10010

The field of Medicine is an ever-changing profession and as new evidence based studies
are conducted, new knowledge is discovered. Ftplectures has made tremendous effort to
deliver accurate information as per standard teaching of medical information at the time
of this publication. However, there are still possibilities of human error or changes in
medical sciences contained herein. Therefore, ftplectures is not responsible for any
inaccuracies or omissions noted in this publication. Readers are encouraged to confirm
the information contained herein with other sources.

ALL RIGHTS RESERVED. This book contains material protected under International
and Federal Copyright Laws and Treaties. Any unauthorized reprint or use of this
material is prohibited. No part of this book may be reproduced or transmitted in any
form or by any means, electronic or mechanical, including photocopying, recording, or
by any information storage and retrieval system without express written permission from
ftplectures.
Future teaching physicians Lectures LLC
Medicine made simple





Title: Abdominal Aortic Aneurysm

Objectives for learning: Define AAA, risk factors, causes, pathogensis, diagnosis
and treatment.

Definition:
Aneurysms are enlarged blood vessels. Aortic aneurysm are mostly infrarenal.

Causes/ Risk factors:
High incidence in males, Age > 50 years, (65-70 years)
- Smoking increases risk of developing AAA- due deposition of atherosclerosis
deposition of fatty plaques inside the wall of the abdominal aorta. This causes the
wall to weaken, and that weakening causes that to balloon out. Atherosclerosis
pathogenesis is due to deposit LDL which the macrophages are getting caught, eat it
up, and become forming macrophages. They are going to cause the proliferation of
the smooth muscle cells, they are going to form a plaque which is going to form in
the walls of this aorta. And that plus smoking, hyperlipidemia, which means high
LDL and low HDL, basically is going to predispose you to develop the aneurysm.

- Hypertension,
- Vasculitis inflammation of small blood vessels in aorta leading to ischemia
and weakening of aortic walls.
- Syphilis
- Marfan syndrome
- Fibrillin deficiency or any connective tissue disorder

Pathophysiology

Clinical symptoms and signs
- patients are asymptomatic except when aorta ruptures
- palpable, pulsating abdominal mass
- ruptured AAA is a life threatening emergency with a mortality of 90%. The
pumping heart pumps all the cardiac output into the abdominal cavity
leading to sever hemorrhagic shock.
Symptoms
- severe abdominal or lower back pain,- patient complain of sudden onset of
abdominal pain.
- Physical Exam may show grey tunre sign- flank ecchymosis
- Cullens sign- Periumbilical ecchymosis- sign of blood in abdominal cavity
Sigs: Triad of AAA- hypotension, abdominal pain, and pulsatile abdominal mass.
Possible syncope if sever hypotension from hemorrhagic shock and inadequate
perfusion to brain.
- nausea, vomiting.

Diagnosis
Ultrasound- 100% sensitive
CT scan only for stable patient with no hemodynamic instability- normal blood
pressure

Treatment
Surgery with synthetic graft
Treat risk factors- Hyperlipidemia with statins, hypertension with beta-blockers,
thiazides, or ACE inhibitors
Advice patient to stop smoking because it increases risk of developing another AAA
Future teaching physicians Lectures LLC
Medicine made simple






Title: Acute Coronary Syndrome

Objectives for learning: Understanding the basic facts about acute coronary
syndromes including unstable angina, NSTEMI and STEMI.

Definitions: Acute coronary syndrome encompasses unstable angina (USA), non-ST
elevation (NSTEMI), and myocardial infarction (STEMI).

Causes/ Risk factors:
Stable angina
Smoking
Diabetes
Obesity

Pathophysiology:
Atherosclerotic plaques in coronary arteries cause the initial pathology. The plaque
can rupture which leads to increased tendency to increased thrombosis.
Thrombosis can lower the blood flow through the affected coronary artery, thus
causing ischemia to the regions distal from the occlusion.
Unstable angina appears on the basis of previously developed stable angina.
Due to seriously decreased diameter of coronary blood vessels caused by
atherosclerotic plaque progression, these patients have increased frequency
of chest pain occurring even during the rest.
Non-STEMI is the type of myocardial infarction which is not big enough to
cause ST-elevation.

Clinical symptoms and signs
Increased frequency of chest pain even during the rest
Prolonged duration of chest pain (>10 minutes)

Diagnosis
EKG can be normal in the case of unstable angina and NSTEMI, but it can also
show ST-depression and T wave inversion due to the ischemia. ST-
depression has to be greater than 0.5 mm and T inversion greater than 2 mm
in order to be significant.
In order to distinguish unstable angina and NSTEMI, cardiac enzymes are
used (CK-MB, Troponin I/T)


Treatment
Morphine
Oxygen
Nitrates
Aspirin (COX-inhibitor which decreases tromboxan A2 and platelet
aggregation)
Beta-blockers (decreasing heart frequency and blood pressure)
Glycoprotein IIb / IIIa inhibitors
Exonoparin (low-molecular heparin)
Coronary catheterization is performed if the patient is not feeling better after
medicamentous treatment.

Lifestyle modifications:
Diet
Exercise
Glucose control
Statins
Stop smoking
Lose weight

Future teaching physicians Lectures LLC
Medicine made simple






Title: Acute Myocardial Infarction (MI)

Objectives for learning: Learning the pathophysiological and clinical features of
acute myocardial infarction, as well as treatment options.

Definitions: Acute myocardial infarction is necrosis of myocardium due to massive
ischemia caused by coronary artery occlusion. It is the most common cause of the
death in the United States (30% mortality).

Causes/ Risk factors:
Acronym: FLASH MD
Family history
Low HDL
Age (men>45, women>55)
Smoking
Hypertension
Male gender
Diabetes

Pathophysiology: Acute myocardial infarction represents the necrosis of some
parts of heart muscle which happens due to the massive ischemia caused by
complete occlusion of the coronary artery responsible for supplying that region.

Clinical symptoms and signs
Chest pain (crushing; radiation to the neck, jaw, and left arm)
Nausea
Vomiting
Diaphoresis
Shortness of breath
Weakness, fatigue
Symptoms last more than 30 minutes.
Atypical clinical picture in:
Diabetics
Elderly
Women
St. post surgery

Diagnosis
EKG changes (ST-elevation, Q-waves (at least 0.04s and 25% of R wave), T-
waves inversion
o I, aVL, V5, V6 lateral wall
o V1, V2, V3 anterior wall
o II, III, aVF inferior wall
o V1, V2 Septal wall
Cardiac enzyme
o CK-MB (increases during the first 4-8 hours; returns to normal values
after 48-72 hours)
o Troponin I/T (increases in the first 3-5 hours; returns to normal
values after 5-14 days)
* CK-MB is an important marker for reinfarction

Complications
Ventricular fibrillation
Reinfarction

Treatment
Morphine
Oxygen
Nitrates
Aspirin antiplatelet (aspirin decreases mortality)
Beta-blockers (reduce mortality)
ACE inhibitors
Statins
Anticoagulant - Low molecular weight heparine
Revascularization
Future teaching physicians Lectures LLC
Medicine made simple






Title: Acute Pericarditis

Objectives for learning: Learning clinical signs, diagnostic techniques, and
treatment options for acute pericarditis.

Definitions: Pericarditis is an inflammation of pericardium.

Causes/ Risk factors:
Idiopathic
Viruses (Coxsackie B, HIV, Echoviruses)
Radiation
Uremia
Acute myocardial infarction (Dresslers syndrome)
Lupus
Amyloidosis
Rheumatoid arthritis
Sarcoidosis
Procainamide, hydralazine, izoniazide (drug induced pericarditis)


Clinical symptoms and signs
Retrosternal chest pain (pleuritic pain which radiates to trapezius or
scapula)
o Leaning forward lowers the pain.
o Swallowing, cough, and lying down increase the pain.
Fever
Pericardial rub

Diagnosis
EKG
o Diffuse S-T elevation
o P-R depression (the most specific for pericarditis)
o T wave inversion
Echocardiogram

Complications
Pericardial effusion
Pericardial tamponade

Treatment
Aspirin
Steroids are given if the patient does not respond well to aspirin
Future teaching physicians Lectures LLC
Medicine made simple






Title: Aortic Dissection

Objectives for learning: Understanding pathophysiology, causes, and clinical
features of aortic dissection, learn how to diagnose it fast and what type of
treatment to apply.

Definitions: Severe hypertensive emergency where the blood flow damages tunica
intima and accumulates in the space between tunica intima and tunica media.

Causes/ Risk factors:
Hypertension
Trauma
CTD Connective Tissue Diseases (Ehlers-Danlos syndrome, Marfan
syndrome)

Pathophysiology: There are two types of aortic dissection type A and type B. In
type A, proximal part of ascending aorta is dissected, so there is a risk of coronary
ischemia because the dissection can reach coronary arteries. In type B, aortic arch is
dissected, and the dissection can prolong distally, down to renal arteries causing
renal ischemia.


Clinical symptoms and signs
Ripping chest pain (anterior in proximal dissection and interscapular in
distal dissection)
Diaphoresis
Aortic regurgitation signs and symptoms
Hemiplegia or hemiparesis (due to ischemic stroke)

Diagnosis
Transesophageal echocardiogram (TEE)
CAT scan (aorta with two lumens)
AP chest X-ray (widened mediastinum)

Complications
Coronary ischemia
Stroke
Renal ischemia
Treatment
Beta-blockers
i.v. Sodium nitroprusside (decrease systolic pressure below 120 mmHg)
Surgical treatment is necessary in type A aortic dissection

Title: Aortic Stenosis

Objectives for learning: Definition, Pathophysiology, Causes/ Risk factors, Clinical symptoms
and signs, Diagnosis and Treatment.

Definition:

Pathophysiology

Stenotic aortic wall causes left ventricular outflow obstruction. Valve being stenotic becomes
unable to push blood to left ventricle. Decrease LV outflow leads to left ventricular hypertrophy
due to excessive amount of force heart has to put to pump the blood. Over a passage of time
LV dilatation and then mitral regurgitation develops. Cardiac output becomes decrease.

Causes/ Risk factors:

Calcification of the bicuspid valve (olderly patients most develop calcification of the valve due
to deposition of calcium)

History of rheumatic fever

Clinical symptoms and signs:

Symptoms:

Asymptomatic for years


With the worsening of obstruction
Syncope
Angina
Dyspnea due to CHF

Signs:

Murmur
Crescendo-decrescendo murmur. Can be heard at second intercostal space and carotid
artery
S4 sound
Parvus et tardus a decreased or delayed carotid stroke

Diagnosis
Chest-x-ray
Echocardiogram (LV and LA enlargement)
Cardiac catherization: It is a definite diagnostic test and gradient and valve area less
than 0.8 is considered normal.

Quiz

1. A 57 year old patient presents with complaints of Syncope, chest pain on physical exertion
and malaise. On examination a murmur is present over second intercostal space. S4 sound
is also heard. He has a previous history of rheumatic fever. What is the likely diagnosis?
A. Aortic stenosis
B. Aortic regurgitation
C. Mitral stenosis
D. Mitral regurgitation

The correct answer is: A

The history, clinical picture and physical examination all point towards the development of
aortic stenosis in this patient. The murmur of aortic stenosis is usually heard at second
intercostal space. S4 sound is prominent here. Rheumatic fever is the most common cause of
aortic stenosis in the adult patient.

In case of aortic regurgitation S3 sound is heard whereas the murmur of aortic regurgitation is
heard at third left intercostal space.

Mitral stenosis is characterized by the loud first heart sound and tapping apex beat. The
murmur of mitral stenosis is heard at the apical region and is low-pitched.

The murmur of mitral regurgitation is best heard at the apex of the heart. Here first heart
sound is soft. The most common cause of mitral regurgitation is mitral valve prolapsed.

2. Which of the given option is related to the aortic stenosis?


A. Diastolic murmur
B. Atrial fibrillation
C. Right heart failure
D. Systolic ejection murmur
The correct answer is: D

Systolic ejection murmur is related to the aortic stenosis. This murmur is the due to turbulent
flow of blood forward across the right ventricular outflow tract, aortic valve, or via the aorta. It
is also related to pulmonary stenosis.

Diastolic murmur has no relation to the aortic stenosis. Diastolic murmurs begin at or after S2
heart sound and remains till at or before S1.

Aortic stenosis does not lead to atrial fibrillation but mitral stenosis can cause the development
of atrial fibrillation.

Right heart failure may result with aortic stenosis but when disease is quite old so best option is
D.

3. A 43-year-old patient presents with confirm diagnosis of aortic stenosis. He has a history of
fever and joint pains. What would be the most likely cause here?
A. Bacterial endocarditis
B. Congenital bicuspid valve
C. Rheumatic fever
D. Marfans syndrome

The correct answer is: C

Aortic stenosis in an adult is usually the result of rheumatic fever. Rheumatic fever is
an inflammatory condition that precedes an infection by the Streptococcus pyogenes. It can
involve the skin, heart, brain and joints.

A congenital bicuspid valve is basically a congenital disease vulnerable to endocarditis and


ultimately develops calcification as well as symptomatic stenosis.

Aortic stenosis is usually seen after the bacterial endocarditis. There must be a history of
development of bacterial endocarditis.

Marfans syndrome can cause aortic stenosis but that adult must have the features of Marfans
syndrome.

4. Crescendo-decrescendo is the murmur found in aortic stenosis. What is its exact location?
A. Fifth intercostal space
B. Second intercostal space and carotid artery
C. In the fourth intercostal space medial to mid-clavicular line.
D. At the apex of the heart

The correct answer is: B

Since Crescendo-decrescendo murmur occurs due to stenosis of aorta, so its location would be
where aorta is present. The correct answer therefore is second intercostal space and carotid
artery. All other options are not correct.

5. Syphilis is the infection which can cause heart disease. What is the most common valve of
the heart involved by it?
A. Mitral valve
B. Pulmonary valve
C. Aortic valve
D. Tricuspid valve

The correct answer is: C

In the later stages of disease, syphilis involves the heart and remains confined to the base of
the aorta. When it involves the wall of the aorta, syphilis leads to loss of the elastic properties
of the aorta and even the formation of aortic aneurysms.

Mitral valve, pulmonary valve and tricuspid valve are not affected by the syphilis.


Title: Atrial Septal Defect (ASD)

Objectives for learning: Definition, Pathophysiology, Clinical symptoms and signs, Physical
Exam, Diagnosis, Complications, and Treatment.

Definition:
The defect between the two atria is called atrial septal defect. There are two types:
Ostium primum: The little hole at the top between the right and the left atrium is called ostium
primum. 80% septal defects are ostium primum.
Septum primum: It is the defect at the lower side between the right and the left atrium.

Causes/ Risk factors:

Pathophysiology
Deoxygenated blood normally comes from the head and the lower side of the body into the
superior vena cava (SVC) and inferior vena cava (IVC) respectively. From here it enters into the
right atrium. Right atrium now has deoxygenated blood which enters into the right ventricle.
Right ventricle contracts during systole and pumps blood into the pulmonary arteries which
now enters into the lung. The pulmonary arteries become smaller pulmonary capillaries which
take oxygen, exchange with carbon dioxide. Now oxygenated blood is taken up by the
pulmonary veins into the left atrium. The left atrium always has oxygenated blood.
In case of atrial septal defect the left atrium cannot pump blood into the left ventricle. The
oxygenated blood is not transferred to the whole of the body through aorta. Thus, oxygen is
not transmitted to the body.
The pressure from the IVC and SVC coming into the right atrium is always at low side. The
pressure in the RV is 25/10 mmHg, in the left atrium is 12 mmHg and in the left ventricle is
130/80 mmHg.

Clinical symptoms and signs
The patients are normally asymptomatic. But as the patients reach to 40 years of age, they
develop:
Dyspnea at exertion
Exercise intolerance
Anemia
Fatigue

Physical Exam
Mid-systolic ejection murmur is heard at pulmonary area.
Wide fixed split S2 sound
Diastolic rumble in the tricuspid region
Irregularly irregular heart rate
No p waves on EKG
Atrial fibrillation

Diagnosis
Transesophageal Echocardiogram (TEE)
Chest x-ray
Electrocardiography (EKG)

Complications
Pulmonary hypertension
Right ventricular failure
Eisenmenger's disease
Paradoxical emboli
Stroke

Treatment
Surgical repair close up the valve
Title: Blood Pressure Regulation I: Baroreceptor Pathway

Objectives For Learning: Mean arterial blood pressure, Normal blood pressure, Pathways
and Neural Pathways.

Mean Arterial Blood Pressure

Mean arterial blood pressure: 2 (diastolic blood pressure) + systolic blood pressure / 3.
Normal blood pressure is less than 120/80 mmHg i.e. systolic blood pressure over diastolic
blood pressure.

Pathways

There are two pathways

Neural pathways fast sympathetic


Hormonal pathways slow Renin-angiotensin, aldosterone pathway

Neural Pathways

When someone is bleeding, it causes decrease in intravascular volume. The blood pressure also
decreases. Normally, the aortic arch and baroreceptors feel the stretch in the walls of the
carotid sinus. They also sense the decrease in blood pressure. Glossopharyngeal nerve of the
herings nerve in the carotid sinus after sensing this sends signals to the brainstem.
Automatically, the decrease in parasympathetic tone is achieved while increase of sympathetic
tone occurs to compensate the decrease in blood pressure. Sympathetic nervous system acts
on the SA nodes and causes:

Increase in the heart rate


Increase in the contractility of cardiac muscles
Increase in stroke volume

All this eventually increases the cardiac output and finally the BP. The sympathetic system also
activates the alpha and beta receptors in the vasculature leading to vasoconstriction and
increase in total peripheral resistance (TPR). Increase in TPR leads to increase in BP.


Future teaching physicians Lectures LLC
Medicine made simple






Title: Cardiac Tamponade

Objectives for learning: Understanding the basics of cardiac tamponade
pathophysiology, clinical features, diagnosis, and treatment.

Definition: Cardiac tamponade is impaired ventricular filling due to excessive
pericardial effusion.

Causes/ Risk factors:
Penetration of the chest
Iatrogenic damage to the atrial or ventricular wall during central venous
catheter placement or pericardiocentesis
Pericarditis
Myocardial infarction

Pathophysiology:
The rate of effusion is important for the development of cardiac tamponade.
If more than 300 mL of fluid are rapidly going into the pericardial space, it
will cause tamponade. On the other hand, in cases of slow filling, the
pericardium has time to stretch so there can be 1.5 L of fluid in pericardial
space before developing a tamponade.
Impaired ventricular function leads to decreased preload, which then leads
to low cardiac output. Consequently, the blood pressure is going to be low.
The pressures in all four cardiac cavities is going to be equal.

Clinical symptoms and signs
Becks triad:
o JVD
o Hypotension
o Muffled heart sounds
Narrowed pulse pressure (due to low stroke volume)
Pulsus paradoxus (blood pressure decrease for >10mmHg during
inspiration)
Tachycardia
Diagnosis
Echo
Chest X-ray
o Increased cardiac silhouette
o Clear lung fields
EKG
o Electrical alternans
Cardiac catheterization
o Increased intrathoracic pressure
o RA pressure is decreased with the loss of Y descent

Treatment
o If the patient is stable and there is no blood just watch
o If the patient is unstable pericardiocentesis
o Surgery if the vasculature walls are damaged
Title: Cardiac Tumors: Myxomas
Objectives for learning: Types Of Cardiac Tumor, Most Common Primary Liver Cancer, Most
Common Liver Cancer, Most Common Brain Cancer


Types of Cardiac Tumor
1. Myxomas
2. Rhabdomyomas


The most common primary cardiac tumor of adults is atrial myxomas. Patients present with
syncope because this atrial myxoma causes ball valve effect in the left atrium. Due to the tumor
blood cannot get into the left ventricle because this tumor seals up the mitral valve, causing a
decrease in the left ventricular and diastolic volume. The cardiac output also decreases,
perfusion also decreases and the brain gets affected, therefore leads to syncope.

In children the most common primary tumor of heart is Rhabdomyomas. Tuberous sclerosis
is the most common disease associated with this tumor.
The most common primary liver cancer is the hepatocellular carcinoma from liver cirrhosis.
The most common liver cancer is the metastasis usually from melanoma or lymphoma.
The most common brain cancer is the metastasis usually from melanoma or lymphoma.


Title: Cardiogenic Shock

Objectives for learning: Definition, Causes, Clinical symptoms and signs, Diagnosis,
Treatment, Intra aortic balloon pump (IABP).

Definition:
Cardiogenic shock is the decrease in cardiac output that leads to decrease in tissue perfusion.
This is due to insufficient circulation of blood owing to failure of the ventricles of the heart to
work effectively.
We know that
Mean arterial pressure = CO x TPR
CO= cardiac output
TPR= Total peripheral resistance
With decrease in cardiac output, mean arterial pressure also decreases. Cardiogenic shock is a
medical emergency.

Causes/ Risk factors


Acute myocardial infarction (most common cause)
Cardiac temponade
Tension pneumothorax
Arrhythmias (ventricular tachycardia)
Massive pulmonary embolism

Pathophysiology

Clinical symptoms and signs


Symptoms:
Hypotension Blood pressure can fall up to 80/60
Oliguria
Tachycardia
Altered mental status

Signs:
Skin appears pale and cold.
Jugular venous distension is found
Pulmonary congestion or edema is present.

Diagnosis
EKG: To see the presence of elevated ST segments, indicating myocardial infarction (MI)
Echo to check cardiac temponade
Chest X-ray to detect the presence of tension pneumothorax
Hemodynamic monitoring:
Swan-Ganz catheter is used to monitor blood flow and heart's function. It measures left atrial
pressure by measuring capillary pulmonary wedge pressure.

Treatment
The initial management is maintaining of
Airway
Breathing
Circulation

Afterwards following treatment is started depending upon the cause.

In case of myocardial infarction the following treatment is given.
Oxygen
Beta blocker
Morphine
Ace inhibitors
Nitroglycerin
NG intubation
Statin
Tissue plasminogen activator (TPA)
CABG
Angioplasty

For cardiac temponade pericariocenthesis is performed.
Thoracotomy is carried out for tension pneumothorax
The drug such as amiodarone is given for cardiac arrhythmias.
Low molecular weight heparin is given to manage pulmonary embolism.

Vasopressors are also given in cardiogenic shock. They tend to increase after and preload and
thus increase the reduced blood pressure. These are:
Dopamine: It increases the renal flow and renal perfusion
Dobutamine: It increases the cardiac output
Norepinephrine or phenolepinephrine: It is used when both dopamine and dubutamine do
not help to raise the BP. Norepinephrine increases contractility, cardiac output and
eventually the blood pressure.

Dont give IV (intravenous) fluids here since they are harmful for the patient with cardiogenic
shock. Left ventricular pressure is already elevated so IV fluid should be avoided.

Intra aortic balloon pump (IABP)
Intra aortic balloon pump (IABP) is sometimes used to boost myocardial oxygen perfusion.
Balloon is placed in aorta. During systole this balloon deflates while during diastole it inflates.
By doing so it increases afterload and cardiac output and consequently the perfusion and
oxygen to the coronary artery also increases and myocardiac oxygen demand decreases.


Quizzes
1. A patient presents with chest pain, difficult breathing and confusion after long trip in an
airplane. His BP was 80/40mmHg. His skin is cold. Jugular vein is distended. Based on the
findings and history what is the likely complication of pulmonary embolism in this patient?
A. Hypovolemic shock
B. Cardiogenic shock
C. Anaphylactic shock
D. Neurogenic shock

The correct answer is B.
The most likely complication pulmonary embolism in this patient is cardiogenic shock. This is
because massive pulmonary embolism leads to the development of cardiogenic shock. It is also
apparent from the condition of the patient. He has developed hypotension, skin is cold and his
jugular vein is also distended, pointing toward cardiogenic shock.

Hypovolemic shock may present wit hypotension, cold skin but there is no chest pain unless
there is a trauma there. Also, in this shock hemorrhage or bleeding occurs. Jugular vein is not
distended. The most important is that pulmonary embolism does not lead to hypovolemic
shock as no blood loss occurs here.

Anaphylactic shock is the characteristic of severe allergic reaction. There may be a number of
signs and symptoms such as itch, swelling, rash, low BP, reduce heart rate, dyspnea, etc. it is
not the complication of pulmonary embolism.

Neurogenic shock is again not the complication of pulmonary embolism. It occurs after spinal
cord injury.


2. In a patient with diagnosis of myocardial infarction and cardiogenic shock, what will be the
most important treatment of choice?

A. Intra aortic balloon pump (IABP)
B. Angioplasty
C. CABG
D. Pericariocenthesis

The correct answer is A.
Intra aortic balloon pump (IABP) is the most effective and important treatment of choice in
patient with myocardial infarction and cardiogenic shock. This is because it increases the
perfusion to the myocardium by increasing the coronary blood flow as well as simultaneously
decreases the myocardium oxygen demand, which is important to prevent re-infarct and tissue
death.

Angioplasty can be used but it is merely used to open up the obstructed vessels.

Coronary artery bypass grafting (CABG) is basically a surgical procedure that is performed to
improve the blood flow. It is performed when there is severe coronary heart disease (CHD).

3. Pericariocenthesis is not performed here and is mainly done in case of cardiac temponade.
A patient presents with cardiogenic shock and myocardial infraction. After resuscitation all of
the following should be administered based except?

A. Beta blocker
B. IV fluids
C. Tissue plasminogen activator (TPA)
D. Morphine

The correct answer is B.
IV fluids should not be administered in the patient with cardiogenic shock. This is because they
are harmful for the patient with cardiogenic shock. They can raise the volume and eventually
left ventricular pressure which is already elevated so IV fluid should be avoided.

Beta blockers are useful in patient with cardiogenic shock and myocardial infarction because
they reduce the size of infarct as well as early mortality when administered early. Beta blockers
also decrease the incidence of development of recurrent ischemia, ventricular arrhythmias, or
reinfarction.

Tissue plasminogen activator (TPA) can be given in patient with cardiogenic shock and
myocardial infarction. It helps to breakdown the thrombosis.

Morphine is also used in myocardial infarction as it reduces the pain.


4. A 18-year-old patient is admitted to the hospital because he presents with confusion, chest
pain, and confusion and shortness of breath after a blunt injury in the chest. On
examination he is pale with cold clammy skin. His BP is 80/60 mmHg. His jugular vein is
distended and urinary output noted is 17ml/hour. A diagnosis of cardiogenic shock is made.
What is the likely cause of cardiogenic shock in this patient?

A. Acute myocardial infarction
B. Arrhythmias
C. Tension pneumothorax
D. Massive pulmonary embolism

The correct answer is C.
History of blunt trauma with chest pain and shortness of breath is the typical symptoms of
tension pneumothorax. It is the result of penetrating injury of the lung causing development of
one-way valve. Tension pneumothorax can further cause the development of cardiogenic
shock, thus it is the most likely cause here.

Acute myocardial infarction does not occur after trauma. Also, its symptoms are different such
as here a person develops chest pain that radiates towards neck and left arm, sweating,
palpitation, etc.

Arrhythmias are the condition in which heart beat is irregular. It is either too fast or too slow. It
is presented with shortness of breath and also does not develop after trauma.

Massive pulmonary embolism cannot be the likely cause of cardiogenic shock here because it
does not develop following a trauma. For it to develop, there should be a history of prolong bed
rest, deep venous thrombosis, a long trip in a car or aero plane, etc.

Title: Coarctation Of Aorta


Objectives for learning: Definition, Classification of Coarctation of Aorta, Clinical Symptoms and
Signs


Definition:
It is the stenosis or narrowing of the aorta.

Classification of Coarctation Of Aorta
Pre-ductual
Post ductual

The most important duct is the ductus arteriosum. It is a little pipe that connects the aorta and
the pulmonary artery during intrauterine life. The ductus arteriosum later becomes ligamentum
arteriosum. If something happens before, it is called preductual coarctation.
If something happens after, post

Preductal coarctation only occurs in infants.
The adult type is the post ductual coarctation. By definition this is a stenosis of the aorta past
the ligamentum arteriosum or stenosis distal to the ductus arteriosus is known as post ductual
coarctation.

Coarctation of aorta is associated with Turner syndrome. Aortic regurgitation eventually
develops. The murmur of coarctation of aorta is the blowing high pitched diastolic murmur.

Causes/ Risk factors:

Pathophysiology

Clinical symptoms and signs
Low blood pressure in lower extremities (hypotension)
High blood pressure in upper extremities (hypertension)
Notching of the ribs and pleural effusion on chest X-rays may be present.
Poor growth

Diagnosis

Treatment

Future teaching physicians Lectures LLC
Medicine made simple






Title: Complications of Acute Myocardial Infarction

Objectives for learning: Identifying acute and chronic complications of acute
myocardial infarction.

1. Chronic Heart Failure
The main symptom is shortness of breath
Therapy:
o ACE inhibitor to decrease blood pressure
o Diuretics (furosemide) to eliminate fluid
It can progress to cardiogenic shock

2. Arrhythmias
Premature Ventricular Complexes (PVCs)
Atrial Fibrilation
Ventricular tachycardia
o Therapy:
Amjodaron
Electrical cardioversion (if hemodynamically unstable)
Ventricular fibrillation
o Therapy
Unsynchronized defibrillation
Paroxysmal Supraventricular Tachycardia (PSVT)
Sinus tachycardia
Sinus bradycardia (atropine if they are dynamically unstable)
Asystole
AV block (pacemaker is needed if there is IIb or III degree AV block)

3. Reinfarction
New ST elevation
o Check CK-MB level

4. Rupture
Occurs due to scar tissue forming 10 days after myocardial infarction
Types:
o Free wall rupture (cardiac tamponade) pericardiocentesis is needed
o Interventricular wall rupture surgery is needed
o Papillary muscles rupture (mitral regurgitation) mitral valve
replacement is needed
o Ventricular aneurysm

5. Pericarditis
Inflammation of pericardium
Dresslers syndrome can develop weeks to months after myocardial
infarction
Treatment:
o Aspirin

6. Ventricular embolism
Can cause stroke


Title: Congenital Heart Diseases

Objectives for learning: Right To Left Shunt, Left To Right Shunt and
Pathophysiology.

Definition:


1) Right to left shunt it means early cyanosis, blue babies
2) Left to right shunt late onset cyanosis also known as blue kids

Right to left shunt occurs in

Tetralogy of Fallot (TOF)


Transposition of great vessels
Persistent truncus arteriosus
Tricuspid atresia
Total anomalous pulmonary venous return

Left to right shunt

Ventricular septal defect (VSD)the most common congenital cardiac defect.


Atrial septal defect (ASD)
Patent ductus arteriosus (PDA)

Pathophysiology
The normal pressure inside the right atrium (RA) is less than 5 mmHg

The normal pressure of the right ventricle (RV) is 25/5mmHg, in the left atrium (LA) is less than
12 mmHg, in the LV is 130/10 and in the aorta is 130/90 mmHg.

The pressure in the RA is less than 5mmHg because this allows blood to go into the heart during
preload because pressure in the veins has to overcome the pressure in the atrium. The pressure
in the RV is 25 mmHg, so to receive blood; pressure in it would be equal or less than the
pressure in the RA during diastole so that blood can easily flow into the right ventricle. It
means when heart contracts the pressure the systole pressure which ejects blood into the
pulmonary artery has to be as high as 25mmHg so that it can get blood into the pulmonary
artery. Also, blood from pulmonary artery enters into the left atrium. The systolic pressure
inside the pulmonary artery is 25 and in the left atrium is less than 12; mean high pressure to
low pressure.
The systolic ejection pressure of 130/10 in the LV pushes out the blood into the aorta.

Initially, blood moves from higher pressure to lower pressure, i.e. it shunts from LV to RV. But
there is still a blood to enter into the systemic circulation. Over time, the wall of the RV gets
thick and thicker the wall the higher the diastolic pressure. Overtime, pressure in the RV
overcomes the pressure in the LV, now we get deoxygenated blood into the RV. Thats why it
takes time and children are called blue kids and a patient is said to develop late onset cyanosis.
Similar happens when blood shunts from LA to RA and patient develops late onset cyanosis.

Same is with the PDA, an open space between the aorta and the pulmonary artery.


Clinical symptoms and signs

Diagnosis

Treatment

Future teaching physicians Lectures LLC
Medicine made simple






Title: Congestive Heart Failure

Objectives for learning: Understanding the pathophysiology of different types of
heart failure, as well as risk factors, clinical features, diagnostics, and treatment
options.

Definition:
Heart failure is a syndrome which appears when heart is not able to maintain
circulation and to provide adequate perfusion to the rest of the body.

Causes/ Risk factors:
Hypertension
Excess salt in intake

Pathophysiology:
Normally, heart receives the blood from the venous side of the body, through the
superior and inferior vena cava (preload). After the blood enters the right atrium
and then right ventricle, the heart muscle stretches, which is the stimulus for the
contraction. That way, the blood is being pushed from the ventricles, which is
called stroke volume. Frank Starling relationship says that if the preload is
increased, the contractility increases too, thus causing increased stroke volume
and consequently increased cardiac output.
Decreased cardiac output causes the activation of compensatory mechanisms,
including carotid sinus baroreceptors, which in turn increases sympathetic
activity (increased heart rate, contractility, increased preload and afterload).
Another, slower compensatory mechanism includes rennin-angiotensin-
aldosterone system (water and sodium retention).
The cause of heart failure can be either systolic or diastolic dysfunction.
o Systolic dysfunction is caused by lowered contractility of some parts of
heart muscle due to myocardial infarction, dilated cardiomyopathy,
myocarditis.
o Diastolic dysfunction appears due to impaired relaxation of heart muscle.
The common causes are hypertrophic ventricular failure (due to
hypertension, aortic stenosis, aortic regurgitation, and mitral stenosis)
and restrictive cardiomyopathy (amyloidosis, sarcoidosis,
hemochromatosis).



Clinical symptoms and signs
Shortness of breath Dyspnea (because of pulmonary congestion)
Orthopnea breathing difficulties when lying on the back
Paroxysmal nocturnal dyspnea
Nocturnal cough
Diaphoresis
Cold extremities
Specific signs of left-sided heart failure include:
Cardiomegaly
S3 gallop
S4
Crackles (rales) in the lungs
Dullness to percussion of the lungs
Specific signs of right-sided heart failure include:
Jugular venous distension (JVD)
Liver congestion and hepatomegaly
Ascites
Peripheral edema
Right ventricular heaves
Nocturia
Diagnosis
Chest X-Ray cardiomegaly, Kerley B-lines, interstitial markings, pleural
effusion (blunting of the costophrenic angle)
Echocardiogram is the most important examination to make, because it
shows whether the systolic ejection fraction is lowered.
ECG findings are not specific.
Treatment
1. Diet restriction (less than 4g of salt/day)
2. Diuretics
o Furosemide first line treatment
o Thiazide second line treatment
o Spironolactone (can cause hyperkalemia!)
3. ACE inhibitors decrease mortality in patients with heart failure by decreasing
preload and afterload. Side effects of ACE inhibitors are: angioedema, cough,
and elevated potassium levels. ACE inhibitors can be replaced with sartans
(angiotensin receptor blockers) in patients with serious cough.
4. Beta blockers (Carvedilol) are shown to slow the progression of heart failure
and decrease mortality.
5. Group IV patients (the terminal stadium of heart failure) should receive
Digitalis in order to improve cardiac output. Hydralazine and isosorbid
nitrate can also be used to regulate the blood pressure in these patients.


Future teaching physicians Lectures LLC
Medicine made simple






Title: Coronary Circulation

Objectives for learning: Define coronary arteries and their topography, and
explain coronary circulation as well as physiology of coronary blood supply.

Definition: Coronary circulation is composed of coronary arteries, the most
proximal branches of aorta. Their purpose is blood supply to the heart muscle.

Anatomy: Left coronary artery (LCA) and right coronary artery (RCA) come off the
ascending aorta as its first branches. LCA splits into two arteries: the circumflex
artery (CFX), which wraps around the left lateral side of the heart and finishes on
the posterior side, and left anterior descending artery (LAD) which travels down the
interventricular septum and finishes at the apex of the heart. RCA travels
downwards giving its branch acute marginal artery (AMA). RCA then curves
around the back of the heart forming posterior descending artery (PDA).

Blood supply:
CFX posterior left ventricle
LAD anterior septum and apex
AMA right ventricle
PDA posterior septum

Physiology:
- At rest, myocardium uses 70% of oxygen contained in the blood that comes from
coronary blood vessels, while during exercise, it extracts 90% of oxygen from the
blood.
- Coronary arteries receive blood only during the diastolic phase of cardiac cycle.
Future teaching physicians Lectures LLC
Medicine made simple






Title: Deep Venous Thrombosis

Objectives for learning: Understanding the causes, risk factors, pathophysiology,
diagnosis, complications, and treatment of deep venous thrombosis.

Definition:
Deep venous thrombosis is the presence of clots in deep veins of legs.

Causes/ Risk factors:
Virchow's triad
o Endothelial damage
o Hypercoagulability
o Stasis
Risk factors:
Congestive heart failure (CHF)
Immobilization
Obesity
Estrogen (use of birth control pills or pregnancy)
Family history of DVT
Varicose veins
Pathophysiology:
Unlike arterial blood vessels, blood flow in veins depends only on muscle
contraction. One way valves enable bringing the blood up towards the heart.
Endothelial damage can be caused by:
surgery
stasis (prolonged rest or travel)
malignancies
age.
Some hereditary diseases can cause hypercoagulability.
Factor V Leiden deficiency leads to increased clotting
Protein C and S deficiency
Antithrombin III deficiency
DVT most often comes from iliac and femoral veins.

Clinical symptoms and signs
Signs and symptoms are very variable from patient to patient. Classic findings
include:
Lower extremity pain and swelling (especially while walking) non-specific,
non-sensitive
Homans sign calf pain with dorsiflexion
Fever

Diagnosis
Order Doppler ultrasound of the lower extremities to determine
compressibility of the veins. It is highly specific and highly sensitive for the
detection of blood clots in proximal parts of extremities, but not in the calf.
The most accurate test is venography.
D Dimer (very specific but only about 50% sensitive)

Complications
Pulmonary embolism - detached clots from deep veins of lower extremities
travel through the inferior vena cava to the heart and are then towards the
lungs. The result is pulmonary embolism.
Big saddle emboli that obstruct pulmonary artery cause right
ventricular failure and arrhythmia and hypoxia.
Postthrombotic syndrome appears due to insufficiency of venous valve
system and the increase in hydrostatic pressure in venous capillaries.
Phlegma cerulea dolens severe leg edema resulting in ischemia which
causes loss of sensitive and motor neural function.

Treatment
Anticoagulant treatment is the most important
Heparin (prolongs PTT)
Varfarin (inhibits vitamin K)
TPA (Tissue Plasminogen Activator)
INR should be maintained between 2 and 3 for 3-6 months.
Greenfield filter is used to prevent pulmonary embolism
Surgery post-management (leg elevation, compression stockings, early
ambulation, pneumatic compression boots.
Title: Einsenmengers Disease

Objectives for learning:


Before studying Einsenmengers Disease, first discuss the diseases that cause right to left shunt.

Ventricular septal defect (VSD)


Atrial septal defects (ASD)
Patent ductus Arteriosus (PDA)

These are the leading cause of development of Einsenmengers Disease. A patient may have
either of these diseases for very long duration and if not corrected, they can lead to the
development of Einsenmengers Disease.

Definition:

Causes/ Risk factors:

Pathophysiology
Usually an uncorrected VSD, ASD and PDA lead to compensatory pulmonary hypertrophy which
further results in progressive pulmonary hypertension which increases pulmonary vascular
resistance. This causes reversing of shunt from left to right to right to left. This causes late
cyanosis, clubbing and polycythemia.

The RA pumps blood into the RV and from there to PA to the lungs. But when there is a space
i.e. ventricular septal defect the pressure in the LV overcomes pressure in the RV. This patient
starts pump blood into the RV. But with the passage of time RV hypertrophy develops. Blood
does not pump into the lungs and causes the patient to become cyanosed.

In case of ASD, lots of blood goes into the pulmonary artery. This causes development of
pulmonary hypertension due to increase blood flow into the lungs. The blood again comes to
RV, causing RV hypertrophy.

In case of PDA, there is an open space between the aorta and the pulmonary artery. This
patient shunts deoxygenated blood into the PA into the lungs. Overtime, vasoconstriction
develops, causing pulmonary hypertension. If it is not corrected the blood come backs into the
aorta, causing cyanosis, polycythemia and clubbing to develops.

Polycythemia

Poly means many


Cythemia means cell

When body senses that it is not getting oxygen, it stimulates the production of erythropoietin
to further stimulate the production of more red blood cells, thus causing polycythemia.

Clinical Symptoms And Signs


Cyanosis
Clubbing

Diagnosis

Treatment


Title: Atrial Fibrillation

Objectives for learning: Definition, Causes, Pathophysiology, Clinical symptoms and signs,
Diagnosis and Treatment.

Definition:
It is the fibrillation or quickering of the atrium.

Causes/ Risk factors:
Pulmonary embolism, COPD
Iatrogenic
Rheumatic heart disease / Mitral regurgitation
Arthrosclerosis
Thyroid (Hyperthyroidism, thyrotoxicosis)
Endocarditis
Sick sinus syndrome

Pathophysiology
Normally, SA nodes fires and causes depolarization of the muscles. When atrium depolarizes an
atrial contraction occurs. Atrium also squeezes at the same time, causing blood to enter into
the ventricles.
But in case of atrial fibrillation, multiple different ectopic nodes start firing. Not only atrium
contracts, but blood stasis also occurs.

Clinical symptoms and signs
Lightheadedness
Syncope
Fast heart beat
Hypertension

Diagnosis
On EKG having atrial fibrillation, check
Rate
Regular or irregular QRS complex
P waves
P:QRS ratio
PR interval
QRS width

In atrial fibrillation, there are no P waves present. The patient heart rate is very variable. The
heart beat is irregularly irregular. No P: QRS ratio since no P waves are present. The normal PR
interval is less than 0.02 seconds but in atrial fibrillation, no P waves are present so PR interval
cannot be determined. QRS complex is usually of 120 milliseconds. In atrial fibrillation QRS
width is less than 120 milliseconds or normal.

Treatment
Patients are given:
Anticoagulants such as warfarin (because of blood stasis and if anticoagulants are not given
patients with AF can develop stroke, mesenteric ischemia, claudication, myocardial
infractions, etc.)
Beta blockers such as methoprolol: These help to decrease the heart rate.
Calcium channel blockers e.g. nefidipine: They slow down the heart rate.
Digoxin: It has a parasympathetic effect, stimulating the vagal nerves which slow down the
firing from SA and AV node and thus allows decreasing the heart rate.


Title: Fetal Red Cell Production

Objectives for learning: Types of Hemoglobin and Organ responsible for Blood formation during
intrauterine life.

Fetal hemoglobin 22
Adult hemoglobin 22


Weeks
First three to ten weeks: Yolk sac is responsible for making blood.
6 weeks: Liver is responsible for erythropoises.
15 and 30 weeks: Spleen takes on this responsibility.
22 weeks to adult: Bones becomes responsible.


Quiz
1. Which organ is responsible for formation of blood during 6 week of intrauterine life?
A. Spleen
B. Bones
C. Yolk sac
D. Liver
The correct answer is D.
Liver is responsible for erythropoises during 6 week of intrauterine life.
Yolk sac is responsible for making blood during first 3 to 10th week of intrauterine life.
Spleen becomes responsible for erythropoises during 15 and 30 weeks of life.
Bones becomes responsible from 22 weeks to onward in adult life.

Which organ is responsible for formation of blood during 25 week of intrauterine life?
A. Spleen
B. Bones
C. Yolk sac
D. Liver
The correct answer is A.
Spleen becomes responsible for erythropoises during 15 and 30 weeks of life.
Liver is responsible for erythropoises during 6 week of intrauterine life.
Yolk sac is responsible for making blood during first 3 to 10th week of intrauterine life.
Bones becomes responsible from 22 weeks to onward in adult life.

During which stage does the yolk sac take part in erythropoises?
A. Does not take part
B. 4th week
C. 22 week
D. 15th week
The correct answer is B.
Yolk sac is responsible for making blood during first 3 to 10th week of intrauterine life. So,
correct option is 4th week here.
Yolk sac does take part in the formation of blood during intrauterine life.
During 15 and 30 weeks of life spleen becomes responsible for erythropoises.
Bones becomes responsible from 22 weeks to onward in adult life.

4. What is the difference between Fetal hemoglobin and Adult hemoglobin with regard to their
structure?
A. Fetal hemoglobin has two alpha and two beta chains.
B. Adult hemoglobin has two alpha and two gamma chains.
C. Fetal hemoglobin has two alpha and two gamma chains.
D. Both are same and have no difference.

The correct answer is C.
Fetal hemoglobin has two alpha and two gamma chains while adult has two alpha and two beta
chains.


Fetal hemoglobin has not two alpha and two beta chains but has two alpha and two gamma
chains.


Adult hemoglobin does not have two alpha and two gamma chains but have has two alpha and
two gamma chains.

Fetal and adult hemoglobins are not same but differ structurally as well as with respect to their
life.

5. Which organ is responsible for formation of blood during adult life
A. Spleen
B. Bones
C. Yolk sac
D. Liver
The correct answer is B.
Bones are responsible for formation of blood from 22 weeks to onward in adult life.
Spleen becomes responsible for erythropoises during 15 and 30 weeks of life.
Yolk sac is responsible for making blood during first 3 to 10th week of intrauterine life.
Liver is responsible for erythropoises during 6 week of intrauterine life.

Title: Heart Blocks
Objectives For Learning: Atrio ventricular nodal block, Types, First degree AV block, Second
degree AV block, and Third degree heart block.

Atrio Ventricular Nodal Block

In AV nodal block there is an impairment of the conduction between the atria and ventricles of
the heart. SA node fires but this discharge does not go beyond the atria to the ventricles.

Types

There are three types of blocks.

1. First degree AV block:

It is always prolonged in case of first degree AV block. Normally, it is less than 200 mili seconds.
But in first degree AV block, the PR interval is greater than 200 mili seconds (greater than 5
boxes). This is because there is a block causing a delay in the conduction between the AV node
down into the ventricle. It is a begin condition and requires no treatment.

2. Second degree AV block

It is of two types

Mobitz type 1: It is also known as Wenckebach. A prolonged PR interval until a p wave fails to
conduct is known as mobitz type 1. It is again a begin condition and requires no treatment.

Mobitz type 2: P waves here fail to conduct but the PR interval is constant. Here AV node is
conducting and the block is actually in the bundle of His. For this reason, constant PR interval
appears. Patients may have palpitations. They need pace maker implantation. This type of
condition can progress into complete heart block.

3. Third degree heart block

It is the absence of conduction of atrial impulses to the ventricles which means there is no
correspondence between P waves and QRS complexes. The atrial conduction is doing
everything independently. There is a complete AV block. There is dissociation between atrial
impulses and ventricular conduction. Complete heart block leads to asystole. Patients suffer
from presyncope episodes. They develop lightheadedness or dizziness. They also develop
ventricular tachycardia and atrial fibrillation. In EKG, atrio-ventricular rates are different from
each other. P waves are present. P: QRS ratio is variable. QRS width is normal. A pace maker is
needed to treat third degree AV block.
Title: Heart Sounds Basics

There are four heart valves

Aortic valve (right upper sternal border)


Pulmonary valve
Tricuspid valve (left lower sternal border)
Mitral valve

The apex is in the 5th and 6th intercostal space.

The normal heart sound is S1 and S2

S1 is the sound heard when the mitral and the tricuspid valve close as the blood flow from
atrium to the ventricle and ventricle starts to squeeze.

S2 heart sound is due to closure of aortic and pulmonary valve and is heard as the ventricular
systole ends and the ventricular diastole begins.

S3 sound is present in case of pregnancy or in congested heart failure. It can be heard in the
apex and in the tricuspid area.

S4 sound is due to the atrium contracting against a non-compliant ventricle.


Future teaching physicians Lectures LLC
Medicine made simple






Title: Hypertensive Emergency

Objectives for learning: Understanding the basics of hypertensive urgency and
emergency, and learn to diagnose them and apply fast and effective treatment.

Definitions:
Hypertension is a medical condition in which blood pressure is higher than
140/90 mmHg in two separate occasions.
Hypertensive urgency blood pressure of 220/120 mmHg or higher without
any end organ damage.
Hypertensive emergency blood pressure of 220/120 mmHg or higher with
end organ damage.
Hypertensive encephalopathy blood pressure of 240/140 mmHg or higher
with neurologic symptoms.
Preeclampsia Hypertensive episodes during pregnancy (140/90mmHg or
higher) with proteinuria and edema of lower extremities.

Causes/ Risk factors:
Risk factors:
Hypertension is the most prevalent in African Americans
Men are more often affected than women
Causes:
Noncompliance to medications
Sympathomimetic drugs
Cushings syndrome
Eclampsia
Pheochromocytoma
Hyperaldosteronism

Pathophysiology:
Patient using can develop hypertensive urgency due to bad compliance to
antihypertensive medication they normally use.
Cocaine, LSD, and phenylephrine users can experience hypertensive urgency
due to sympathomimetic effects of these drugs.
Increased blood pressure damages the endothelium of blood vessels. That causes
the deposition of proteins in the walls of the blood vessels, thus causing basement
membrane thickening. That thickening narrows the walls of blood vessels causing
ischemia by decreasing blood flow through the blood vessels.


Clinical symptoms and signs
Head
o Headaches
o Blurry vision (due to papiloedema)
o Altered mental status
o Weakness in arms or legs, numbness, tingling, etc.
o Retinal hemorrhages
Chest
o Chest pain
o Shortness of breath
o Pulmonary crackles/rales
o Jugular venous distension
o S3
Kidneys
o Anuria
o Hematuria
o Increased creatinine
Legs
o Edema


Diagnosis
It is necessary to take good history.
Physical exam:
o If there is altered mental status, rule out the other possible causes in
order to blame hypertension.
o Full neurological exam
o Fundoscopic exam
Laboratory techniques:
o ECG
o Electrolytes
o Creatinine
o BUN
o WBC count
o LFTs
o Proteins and blood in urine
Iamging:
o CT scan of the head
o Chest X-Ray


Treatment
The aim is to decrease the blood pressure by 25% in first 1 2 hours.
*If patients blood pressure is 240/140 mmHg, the pressure after 1 2 hours should
be 180/90 mm Hg.
Medication:
Beta blockers (contraindicated if the patient used drugs)
Sodium nitroprusside
Fenoldopam
Hydralazine
Nitroglycerin

Title: Hypovolemic Shock

Objectives for learning: Definition, Causes, Pathophysiology, Signs and Symptoms Stages of
hypovolemic shock, Diagnosis and Treatment

Definition:
Low blood volume leads to decrease cardiac output.

Causes/ Risk factors:


1. Hemorrhagic

Trauma
GI Bleed
Retroperitoneal bleed

2. Non hemorrhagic
Burns
Vomiting
Massive watery diarrhea
Third space-- Ascites in massive liver cirrhosis
Low albumin (hypoalbuminemia)

Pathophysiology
Low blood volume causes low cardiac output. The parameters of hypovolemic shock include
decreased cardiac output, increased total peripheral resistance (TPR) and decreased pulmonary
wedge pressure.

Clinical Symptoms And Signs


Hypotension
Cold, clammy skin

Diagnosis
CVP (central venous pressure)
Pulmonary capillary wedge pressure is low.
Systemic vascular resistance is high.

Stages Of Hypovolemic Shock


Stage 1: It is characterized by 10 to 15% (<75ml) loss of blood. The normal cardiac output is
equal to 5L (5000ml) so all the parameters such as pulse, systemic blood pressure, respiration,
capillary refill, CNS dysfunction and urine output are not affected but normal.

Stage 2: Here 20 to 30 % (75 to 1500ml) blood is lost. Here systemic BP is normal, but pulse is
increased and heart rate is high, while all other parameters are decreased.

Stage 3: Here 30 to 40% (1500 to 3000 ml) blood loss occurs. Here blood pressure decreases
severely, heart rate increases (due to reflex tachycardia), pulse pressure and capillary refill both
decrease. Lactic acidosis ensues. A patient is confused and his urinary output becomes less than
20ml/hour (patient develops severe oliguria and almost going to develop anuria).

Stage 4: Here greater than 40% blood loss occurs (i.e. > 2000ml). The affect person is lethargic.

Treatment

Maintain airways, breathing and circulation. For maintaining airways incubate the patient.
Manual pressure over bleeding is important to stop bleeding.
Pass NG tube to avoid development of aspiration pneumonia
IV fluids Normal saline.

Quizzes
1. A patient presents with confusion and lethargy. He sustains a trauma on his abdomen. On
examination his BP is 80/50. His skin is cold and clammy. The jugular venous pressure is
normal. What is the most likely diagnosis?
A. Hypovolemic shock
B. Cardiogenic shock
C. Aortic aneurysms
D. Neurogenic shock

The correct answer is A

The most likely diagnosis is hypovolemic shock. This is because patient suffers from a trauma on
his abdomen so it may have cause retroperitoneal bleed. For this reason, his BP has fallen and
his skin is cold.

Since there is no complaint of chest pain, so the cardiogenic shock is unlikely here. The jugular
venous pressure must be elevated in case of cardiogenic shock.

Aortic aneurysm is a condition in which aorta is dilated to greater than 1.5 times than its normal
size. The symptoms of this problem appear when aneurysm is ruptured. There may be
abdominal and back pain. Aortic aneurysm rupture is a serious medical condition.
Neurogenic shock results when there is injury to the spinal cord, affecting the sympathetic
system. Skin here is warm. Bradycardia is also present.

2. A patient is admitted in the hospital after a diagnosis of hypovolemic shock is made. What is
the most common cause of this shock?
A. Burns
B. Injury to spinal cord
C. Fluid shift
D. Blood loss

The correct answer is D.

The most common cause of hypovolemic shock is blood loss. Loss of blood leads to immediate
volume depletion. This depletion further affects cardiac output which becomes reduced.

Burns may lead to development of hypovolemic shock but is not as common as blood loss is.

Injury to spinal cord is the most common cause of neurogenic shock.

Fluid shift can also cause hypovolemic shock since it leads to decrease in blood pressure to
severe extent but it occurs rarely.

3. A patient is presents with severe bleeding and confusion after a trauma. His BP is
80/40mmHg. His heart rate is 120beats/min. He is admitted in the hospital and necessary
resuscitation is given. It is however found that his urinary output is 15ml/ hour. What is the
most likely stage of hypovolemic shock in this patient?
A. Stage 1
B. Stage 2
C. Stage 3
D. Stage 4

The correct answer is C.

The most likely stage of hypovolemic shock in this patient based on his clinical presentation is
stage 3. In this stage 30 to 40% blood loss occurs, leading to severely dropping of blood
pressure and urinary output. Heart rate however is increased. A person develops lactic acidosis
and consequently altered mental status and confusion. In this patient urine output is markedly
reduced.

In case of Stage 1 person does not have above mentioned symptoms. He may be asymptomatic
because here only 10 to 15% blood loss has occurred.

Stage 2 is characterized by 20 to 30 % loss of blood. Here systemic BP is normal, but heart rate
is high.

Stage 4 is a severe form of hypovolemic shock characterized by greater than 40% blood loss. A
person may be unconscious and anuric.


Title: Infective Endocarditis

Objectives for learning: Definition, Causes, Classification, Complications Of Infective


Endocarditis, Clinical symptoms and signs, Diagnosis and Treatment

Definition:
Infection of the heart valve is called infective endocarditis.

Causes/ Risk factors:


Bacteria

Classification
1. Acute Endocarditis (less than 6 weeks): The culprit is staphylococcus aureus (more common
in IV drug abusers)
2. Subacute Endocarditis is caused by:
Streptococcus viridians, most prone to dentistry procedures
Enterococcus (bacteria arise from GI tract)

3. Native Valve Endocarditis


Streptococcus viridans
Haemophilus
Actinobacillus
Cardiobacterium
Ekenella
Kingella

4. Prosthetic Endocarditis
The bacteria responsible here are:
Staphylococcus epidermidis (60 days of surgery)
If more than 60 days then staphylococcus aureus is responsible

In blood cultures if streptococci bovi appears positive then it may probably be due to colon
cancer

Complications Of Infective Endocarditis


Cardiac failure
Glomerulonephritis
Mycotic abscess

Pathophysiology

Clinical Symptoms And Signs


Fever
Roth spots (retinal lesions due to vasculitis)
Oslers nodes (painful nodes in the pads of the finger or the toes due to vasculitis)
Murmur
Jane way lesions (lesions on the palm and the soles due to emboli)
Anemia
Nail bed hemorrhages/ Splinter hemorrhages
Emboli

Diagnosis
Blood culture
Transesophageal echocardiogram

Treatment
Empiric Treatment: Start with vancomycin and gentamycin and give intravenously before the
results of blood culture come.

Once the blood cultures come positive for particular organism, such as for streptococcus
viridians then give penicillin. But if a person is allergic to penicillin then switch to ceftriaxone
and gentamycin.

If person is IV drug abuser, then given antibiotic against staphlococcus aureus. Nafcilin for 4
weeks is given plus gentamycin for 5 days

In case of Methicillin-resistant Staphylococcus aureus (MRSA) give vancomycin for 6 weeks.

And if blood culture comes positive for enterococcal bacteria give penicillin/ampicillin (for 4 to
6 weeks)

If a person is allergic to penicillin then administer vancomycin + gentamycin (for 4 to 6 weeks).


Quiz

1. A 15-year-old patient presents with fever and joints pain for 3 day. A patient also complains
of night sweat. On examination she is pallor. Splinter hemorrhage and painful nodules on
the fingers pad are found. What is the most likely diagnosis?
A. Polymyalgia Rheumatica
B. Atrial Myxoma
C. Reactive Arthritis
D. Infective endocarditis

The correct answer is D.

The most likely diagnosis is infective endocarditis. It is characterized by low grade fever, Oslers
nodules, anemia and splinter hemorrhage. In addition, Jane way lesions and Roths spots are also
present. It is a condition of the inflammation of the valves of the heart.

Polymyalgia rheumatica (PMR) is basically a syndrome of unidentified etiology. It usually


occurs in adults and is characterized by muscle pain frequently of the shoulder girdles and the
hip. A patient usually complains of morning stiffness that lasts for more than one hour.

Atrial Myxoma is a tumor of the heart. It is a benign tumor. Fever, joint pain, shortness of breath
and weight loss occur. Painful nodules are not found here, so it is not a correct diagnosis.

Reactive arthritis is an autoimmune condition which is the result of an infection. It is


characterized by conjunctivitis, urethritis and arthritis. This is not correct as patient has no such
complaints.

2. Which of The following is not the cutaneous involvement of infective endocarditis?


A. Janeway's lesions
B. Skin petechiae
C. Roth's spots
D. Osler's Nodules

The correct answer is C.

Roth's spots are not the cutaneous involvement of infective endocarditis. These are basically the
retinal lesions due to vasculitis.

Skin petechiae are usually found in patient with infective endocarditis.


Janeway's lesions are one of the cutaneous involvements of infective endocarditis. These are
lesions on the palm and the soles and are due to emboli.

Osler's Nodules are again the cutaneous manifestation of this condition i.e. infective
endocarditis. These are painful nodes present in the pads of the finger or the toes due to
vasculitis.

3. A person with infective endocarditis has a history of using drugs for pleasure. Which
antibiotic is best for this patient?
A. Nafcilin
B. Vancomycin
C. Amoxacillin
D. Rifampicin

The correct answer is A.

Nafcilin is the drug of choice for the person who is a drug abuser and also suffered from
infective endocarditis.

Vancomycin is usually preferred for those who are allergic to penicillin.

Amoxacillin has no such role in case of infective endocarditis.

Likewise, rifampicin is not the drug used for patients with infective endocarditis.

4. Which of the following statements concerning infective endocarditis is not right?


A. Empiric treatment of infective endocarditis includes vancomycin and gentamycin
B. Splenomegaly is found to occur commonly in acute infective endocarditis than the sub
acute one.
C. Glomerulonephritis is one of the complications of infective endocarditis
D. Infective endocarditis takes place within 2 weeks of bacteremia

The correct answer is B.

The incorrect option regarding infective endocarditis is A. Splenomegaly is not commonly found
in case of acute infective endocarditis. It is rather present in sub acute infective endocarditis.

Empiric treatment of infective endocarditis includes vancomycin and gentamycin. This treatment
is generally started before the report of blood cultures come.

Clinical manifestations of this condition usually take place within 2 weeks of the provocative
bacteremia in approximately 80% of cases.
Glomerulonephritis is one of the complications of infective endocarditis.

5. All of the following are responsible for native valve endocarditis except

A. Streptococcus viridans
B. Streptococcus viridians
C. Haemophilus
D. Ekenella

The correct answer is B.

Streptococcus viridians do not cause native valve endocarditis instead infection by these bacteria
occurs following a dentistry procedure.

Streptococcus viridians are responsible for native valve endocarditis. 55-65% cases of all native
valve endocarditis are caused by viridans streptococci.

Haemophilus and Ekenella both are the cause behind native valve endocarditis.
Future teaching physicians Lectures LLC
Medicine made simple





Title: Introduction to hypertension

Objectives for learning: Causes, risk factors and pathophysiology and
complications of hypertension.

Definition: Hypertension is high blood pressure (>140/90). Essential hypertension
is high blood pressure without identified cause (95% of all patients). Secondary
hypertension has known causes (5% of patients).

Causes/ Risk factors:
Risk factors:
Age
Gender (men have higher risk than women)
Race (African Americans have higher risk of hypertension)
Obesity
Sedentary lifestyle
Increased sodium intake (>4g/day)
Alcohol

Causes:
Secondary hypertension
o Renal causes
Renal artery stenosis
Chronic renal failure
Polycystic kidney disease
o Endocrine system
Hyperthyroidism
Hyperaldosteronism
Hyperparathyroidism
Cushing syndrome
Pheochromocytoma
o Medications
Oral contraceptives
Decongestives
NSAIDs
TCAs
o Coarctation of the aorta
o Illegal drugs
Cocaine
o Sleep apnea
o Birth control pills

Pathophysiology:
Decreased perfusion through the renal artery stimulates rennin-angiotensin-
aldosterone system, thus increasing peripheral vascular resistance and blood
pressure.
Hyperthyroidism increases metabolic rate and consequently the blood
pressure.
Hyperaldosteronism increases sodium reabsorption and causes
hypernatremia, thus increasing intravascular volume and blood pressure.
Patients with Cushing syndrome have excess amount of cortisol which
activates adrenal medulla to produce more norepinephrine and epinephrine,
thus increasing blood pressure.
In pheochromocytoma, high amounts of norepinephrine and epinephrine are
produced due to tumor of adrenal medulla.

Decongestives are intended to make a local vasoconstriction, but also have
impact on systemic blood pressure increase.

NSAIDs block COX2, thus blocking the synthesis of vasodilatatory
prostaglandins.

Cocaine inhibits the reuptake of norepinephrine, thus increasing its blood
concentration and blood pressure.

Sleep apnea causes respiratory acidosis which provokes hypoxia. Hypoxia
leads to hypoxic vasoconstriction in the lungs which leads to pulmonary
hypertension and eventually high blood pressure.

Increased systemic vascular resistance increases the afterload, so the heart has to
work much harder, which leads to left ventricular hypertrophy. Over time, heart
function becomes weaker leading towards dilation of the heart and heart failure.

High blood pressure accelerates arteriosclerosis by damaging endothelium of blood
vessels.

Complications
Cardiac complications
o Coronary artery disease (myocardial infarction)
o Left ventricular hypertrophy and heart failure
o Stroke (hemorrhagic), TIAs, ischemic stroke
o Aortic dissection
o Peripheral arterial disease
Eye changes
o Papilloedema
o Retinal hemorrhages
Kidneys
o Nephrosclerosis
o Renal failure

Title: Kawasakis Disease
Objectives for learning: Clinical Features, Complications, and Treatment

Definition

Causes/ Risk factors:

Pathophysiology

Clinical Features

Conjunctivitis
Rash all over the body (desquamating rash)
Adenopathy (cervical lyphmadenopathy)
Strawberry tongue
Hands and foot (swelling, erythema and peeling)
Burn
Uncontrolled fever for more than 5 days

Complications

Predisposed to coronary artery aneurysm


Myocardial infraction

Diagnosis

Treatment

Intravenous immunoglobulins (IVIG) + aspirin


Title: MITRAL REGURGITATION


Objectives for learning: Definition, Causes/ Risk factors, Pathophysiology, Symptoms, Diagnosis
and Treatment.

Definition:
Blood returns into left side of the heart into the left atrium from the pulmonary vein right back
into left ventricle into the aorta to the rest of the body.
When mitral valve closes it gives S1. The blood flows out through the aorta during systole and
during diastole mitral valve opens up to allow blood to return back to the heart but at the same
time aortic valve and pulmonary valve both close and it gives S2. During systole both tricuspid
and mitral valve close to give S1 sound.

During systole mitral valve blow up and so blood goes back to the left atrium, increasing the
pressure inside it, causing decrease in cardiac output, hypertension and cardiogenic shock.

Causes/ Risk factors:
Acute causes: Endocarditis, S.aureus infection, Myocardial infarction (rupture of papillary
muscles)
Chronic Causes: Rheumatic heart disease, Marfan syndrome, and cardiomyopathy.

Pathophysiology
LA pressure increases here. The size of left atrium is normal but blood now backs up, back to
the pulmonary vein, pulmonary capillaries, pulmonary edema, congestion and eventually
pulmonary hypertension.

Clinical symptoms and signs
Dyspnea
Palpitations
Proximal nocturnal dyspnea
Pulmonary edema

PE: Holosystolic murmur

Diagnosis
Chest x-ray shows dilated left ventricle and pulmonary edema
Echo show presence of MR, dilated left ventricle and decreased left ventricular function


Treatment
Medical therapy: It is started with afterload reduction medications such as ACE inhibitors (such
as lisinopril)
Decrease salt intake

Digoxin

For arrhythmias give CCB (calcium channel blocker) to treat atrial fibrillation (AF)

Anticoagulation therapy (patient may have AF)

Surgical Treatment: Patient needs a mitral valve replacement or mitral valve repair.

Quiz

1. A patient presents with complain of attacks of severe shortness of breath and coughing at
night. The first heart sound appears soft while the apex beat is laterally displaced. There is
a murmur appeared following first heart sound and is of high-pitched. Chest x ray shows
enlargement of the left atrium and the left ventricle. What is the most probable diagnosis?
A. Mitral regurgitation
B. Mitral stenosis
C. Tricuspid regurgitation
D. Tricuspid stenosis

The correct answer is: B

The most probable diagnosis is Mitral regurgitation. This is characterized by the post nocturnal
dyspnea, orthopnea and palpitations. Holosystolic murmur appears following first heart sound
and is of high-pitched. Chest x ray shows enlargement of the left atrium and the left ventricle.

Mitral stenosis is a condition characterized by the narrowing of the mitral valve orifice. It is
presented with the same symptoms as that of mitral regurgitation. Tapping apex beat is present
with very loud first heart sound. Chest X ray shows left atrial enlargement.

Tricuspid regurgitation is the consequence of problem within the tricuspid valve. The symptoms
include those of right-sided heart failure, such as edema, ascites, jugular venous distension
and hepatomegaly. Jugular venous pressure is found to be elevated. Echo shows the presence
of enlargement of right ventricle and right atrium.

Tricuspid valve stenosis is a disease of the valves of the heart which results due to narrowing of
the tricuspid valve orifice. An abnormal pulse is felt in the jugular vein within the neck during
a physical examination.

2. When does the mitral regurgitation take place?


A. During systole
B. During diastole
C. During both systole and diastole

The correct answer is: A

The mitral regurgitation takes place during systole. During systole mitral valve being abnormal
could not propel blood to the left ventricle and so blood goes back to the left atrium, increasing
the pressure inside it with subsequent consequences.

The option B and C are not correct since during diastole pressure exerted on the walls of the
arteries are not enough to proper blood from left atrium to left ventricle.

3. A 46 year old man with severe mitral regurgitation has no symptoms. His left ventricular
ejection fraction is approximately 45% while an end-systolic diameter index is about 2.9
cm/m2. What would be the most suitable treatment in this patient?

A. No treatment
B. Mitral valve replacement or repair
C. ACE inhibitor therapy
D. Diuretic therapy and digoxin

The correct answer is:B

A zurgical treatment is recommended in case of severe mitral regurgitation even if the patient
is asymptomatic. This is because when the left ventricular ejection fraction falls down below
60% it may cause a progressive dysfunction of left ventricle.

If no treatment is given patient may develop cardiac failure and even death of the person
ensues.

ACE inhibitor therapy is of no value in case of asymptomatic patient. It is however used when
patient with mitral regurgitation develops hypertension.

Diuretic therapy and digoxin are indicated when there is presence of hypertension and
arrhythmias respectively. Since this patient is asymptomatic therefore both drugs are not used
here.

4. What is the type of murmur found in mitral regurgitation (MR)?


A. Pansystolic murmur
B. Presystolic murmur
C. Holosystolic murmur
D. Holodiastolic murmur

The correct answer is: C.

In mitral regurgitation Holosystolic murmur is present. It is a high-pitched murmur found at the


apex. It starts from the end of S1 and remains till the beginning of S2.

Pansystolic murmur is although found in MR but it is also present in other conditions of the
heart. It starts from the beginning of S1 and remains till the end of S2.

Presystolic murmur is present in case of mitral stenosis and appears between the A sound and
S1.

Holodiastolic murmur begins from the end of S2 and remains till the beginning of S1 and is not
present in MR.

5. A 55 years old patient presents with difficulty in breathing for one month. He also suffers
from apprehension. He gave a history of severe fever and formation of lesions on the hand
and fingers. A diagnosis of mitral regurgitation was made based on the clinical examination
and radiological results. What is the likely cause of mitral regurgitation in this patient?
A. Infective endocarditis
B. Myocardial infarction
C. Rheumatic heart disease
D. Marfan syndrome

The correct answer is: A.

Although all the options can cause mitral regurgitation but if we see that this patient gives
history of fever as well as lesions on hands and finger (might be Roth's spots or Osler's nodes),
it is then infective endocarditis.

Myocardial infarction is a serious condition presents with chest pain, dyspnea, sweating and
palpitation. Therefore, this is not correct with regard to this patients scenario.

Rheumatic heart disease is usually more common in children. It not only affects the heart, but
also the joints and the central nervous system. It is the result of rheumatic fever caused by a
preceding infection by group A streptococcal.

Marfan syndrome is basically a genetic disorder which is diagnosed earlier. This syndrome tends
to affect the skeletal system. People with this disorder are abnormally tall and have
long limbs and fingers.
Title: Neurogenic Shock

Objectives for learning: Definition, Causes, Pathophysiology, Parameters of Neurogenic
Shock, Clinical symptoms and signs and Treatment.

Definition:
Neurogenic Shock is defined as the absence of sympathetic tone leading to systemic
vasodilatation. However, there is an unopposed vagal nerve activity.

Causes/ Risk factors:


Spinal cord Injury (It causes loss of sympathetic tone and unopposed vagal activity vagal nerve
tone leads to hypotension).

Pathophysiology
Systemic vasodilatation causes decrease in systemic vascular resistance and hypotension
(80/40). The end result is bradycardia with heart rate of 20beats/minute.

Parameters of Neurogenic Shock


Decreased cardiac output, decreased total peripheral resistance (TPR) and decreased
pulmonary capillary wedge pressure.

Clinical symptoms and signs


Hypotensive
Bradycardia as sympathetic system is loss
Warm skin
Urine output might be low or normal

Diagnosis

Treatment
Maintain Airway, breathing and circulation (ABC)
Mobilize spine (cover on neck)
IV fluids is the mainstay of therapy
Also start domapine since it improves cardiac contractility and thus the perfusion is
enhanced.
Administer dobutamine to increase cardiac output.
For braydcardia give atropine
For neurodeficit give methylprednisolone
Try to maintaining their body temperature
Call neurosurgery department, orthopaedics and trauma surgeons

Quiz

A patient sustains an injury in the back. Now he is presented in the night with decrease urinary
output and lethargy. He tells that he has urinated once in the morning up till now. He has been
urinating twice a day only for 2 days. On examination his temperature is 99F. BP is
90/60mmHg. His heart rate is 50 beats/min. What is the mainstay of therapy?

A. IV fluids
B. Dobutamine
C. Spine mobilization
D. Antibiotics

The correct answer is A.

This patient is basically suffering from neurogenic shock, so here IV fluids are the mainstay of
therapy.

Dobutamine is no doubt very important but it only works to enhance the cardiac output
whereas IV fluids increase blood volume and also correct dehydration.

Spine mobilization is important too to prevent further trauma but it alone wont work.

Antibiotics are beneficial and mainstay of therapy when there is a septic shock. In this case,
antibiotics will be given only when there is a suspicion of infection.

A 36-year-old injured male is presented in emergency with warm extremities, increased


respiratory rate, oliguria and rapid pulse. On examination his B.P. is 70/40mmHg, pulse is
140/min and there is bluish discoloration of tip of nails and tongue. Which kind of shock is
developed in this patient?
Hemorrhagic Shock

A. Septic shock
B. Neurologic shock
C. Anaphylactic shock
D. Hypovolemic shock

The correct answer is B.


The clinical findings go with the condition of neurogenic shock. Since patient is injured, it is
possible he has got an injury to his spinal cord. This causes loss of sympathetic tone and
systemic vasodilation due to unopposed vagal activity.

Septic shock occurs when there is a history of infection. In this condition peripheries are usually
cold.

Anaphylactic shock is due to severe allergic reaction and is manifested with a rash, itching,
swelling, low BP and shortness of breath.

Hypovolemic shock is the result of bleeding or hemorrhage anywhere from the body. Although,
this patient is injured, injury can cause internal bleeding but his peripheries are warm instead of
being cold.

A patient has devolved neurogenic Shock shortly after getting an injury to the spinal cord. How
will he be presented clinically?

A. Cold & clammy peripheries


B. Increased heart Rate
C. Increased total peripheral resistance (TPR)
D. None of these

The correct answer is D.

The correct option is D. The periphery is usually warm because of systemic vasodilatation.
Heart rate is decreased due to loss of sympathetic activity and also total peripheral resistance
(TPR) is decreased.
Title: Patent Ductus Arteriosus

Objectives for learning: Definition, Pathophysiology, Murmur of PDA, Clinical Symptoms and
Signs, Treatment.

Definition:
Patents means opened
Duct means artery

During fetal development, there is an open space between the pulmonary artery (PA) and the
aorta. It is known as PDA. Blood actually gets shunted from PA into the aorta because lungs are
not developed.

Causes/ Risk factors:

Pathophysiology
Normally blood goes to the right atrium to right ventricle and that blood shunts through
pulmonary artery. The pulmonary artery shunts blood to the lungs which comes back to left
atrium to aorta to rest of the body.

When the baby borns, and takes deep breathe, the resistance inside the lungs decreases and they
expand and opened up. Afterward, there is no need of ductus to shunt blood. It becomes
ligamentum arteriosum.
The problem arises when aorta starts to work and pumps blood into the circulations and the lungs
expand as the baby borns, the resistance inside the lungs decreases, the blood starts to shunt from
left aorta into the pulmonary circulation because pressure in aorta is very high i.e. blood now
shunts from left to right (aorta to pulmonary artery). Now the right ventricle has to pump blood
against higher pressure. Thus, a patient with patent ductus arteriosus develops right ventricular
hypertrophy because the pressure gradients of the right ventricle are higher so to pump blood
against the higher pressure of PA.
As the blood enters into the lungs, they develop vasoconstrictions inside the lungs and therefore
increasing the pulmonary pressures.

Overtime the pulmonary vascular hypertension causes pulmonary vascular sclerosis inside the
pulmonary vasculature. When the pressure inside the RV is high enough, the patients will now
be able to reverse this flow and starts to shunt this deoxygenated blood from right side to left
side. The deoxygenated blood mixes with oxygenated, causing the patient to become cyanosed
and exhausted.
Murmur of PDA

The murmur of PDA is machinery murmur.

Prostaglandins (PGE) keep PDA open.

Clinical Symptoms And Signs


Symptoms of cyanosis in the lower extremities

Diagnosis

Treatment
It is important to close this PDA. Indomathacin helps to close this PDA.
Future teaching physicians Lectures LLC
Medicine made simple






Title: Peripheral Vascular Disease - Atherosclerosis

Objectives for learning: Learning the process of atherosclerotic plaque formation
and its consequences.

Definition: Atherosclerosis is a peripheral vascular disease which manifests with
creation of atherosclerotic plaques inside the walls of blood vessels.

Causes/ Risk factors:
Hyperlipidemia
Bad eating habits

Pathophysiology: Damage of the endothelial wall causes the migration of
macrophages. Macrophages then accumulate LDL and form so called foam cells.
That way, fatty streak is formed in the wall of the blood vessel. Platelets send signals
to fibrous cells and smooth muscle cells to migrate from tunica media to tunica
intima producing a fibrous plaque, which then progresses to atheroma. Narrowed
blood vessels than cause organ ischemia. Abdominal aorta is the most common
place for artheroma formation.

Clinical symptoms and signs
Pain due to ischemia
Claudications

Complications:
Thrombosis
Myocardial infarction
Stroke

Future teaching physicians Lectures LLC
Medicine made simple






Title: Prinzmetals Angina

Objectives for learning: Understanding Prinzmetals angina clinical features and
treatment options.

Definitions: Transient coronary vasospasms of coronary arteries.

Pathophysiology: Coronary arteries affected by Prinzmetals angina usually already
have thrombosis occluding up to 75% of their lumen. Transmural ischemia is
present.

Clinical symptoms and signs
Chest pain

Diagnosis
Transient ST-elevation
Coronary angiography

Treatment
Calcium channel blockers
Nitrates
Title: Rheumatic Heart Disease

Objectives for learning: Definition, Causes, Pathophysiology, Clinical symptoms and signs,and
Diagnosis


Definition:

Rheumatic Heart Disease is the consequence of the pharyngeal infection.

Causes/ Risk factors:
Group A beta hemolytic Streptococcus

Pathophysiology

This is a type II immune mediated hypersensitivity. Patients with this disease develop a
murmur. They have vegetation and fibrosis. The most affected valve is the mitral valve. In case
of acute disease the antibodies of the M protein of the organism destroy the valve. The
antibodies come and bind to M protein of the organism. They then both attack and destroy the
valve causing an inflammation.

In case of acute inflammatory response on mitral valve, the patient suffers from mitral
regurgitation. These protein attack the glycoproteins of the antigen present on the valve.

In case of chronic disease, fibrosis of the mitral valve occurs causing stenosis of the mitral valve.
Not only mitral valve gets affected but also aortic valve can be affected. Likewise, tricuspid
valve can also get affected. However, mitral and the aortic valve get affected more as compared
to others.

Patients with rheumatic heart disease also develop myocarditis.

Clinical symptoms and signs
Fever (101.2 F)
Erythema marginatum (a red margin rash)
Valvular damage (patients develop murmur due to vegetation of the valves. The most
affected valve which is affected is the mitral valve)
Erythrocyte sedimentation rate is very high
Red hot joints (joints painmigratory polyarthritis)
Sub-cutaneous nodules (Ashoff bodies: These are the granuloma with histocytes with giant
cells)
Saint vitus dance or Sydenham's chorea

Sydenham's chorea is due to the CNS pathology. The patient has an immune reaction. The
antibodies bind the neurons in the brain and thus affect the caudate nucleus and subthalamic
nuclei. Caudate nucleus is important in the body movement.

Diagnosis

ASO: antistreptolysin O titers

Treatment

Title: SEPTIC SHOCK

Objectives for learning: Definition, Causes/ Risk factors, Pathophysiology, Complications,
Symptoms, Diagnosis and Treatment of septic shock.


Definition:
The sepsis pathway
Systemic inflammatory response syndrome (SSRI) (i.e. inflammation and source) sepsis (one
organ failure) severe sepsis (sepsis plus end organ damage) septic shock >2 MODS

Systemic inflammatory response syndrome is defined as fever of more than 38C and higher
heart rate.
Sepsis is the systemic inflammatory response syndrome and the presence of a known
infection.
Septic shock is persistent hypotension despite giving IV fluids or vasopressors. It is the most
common cause of death in ICU.

Causes/ Risk factors:
Bacterial infection by bacteria such as
E.Coli
Klebsialla
Staph aureus
Pseudomonas

Pathophysiology
As the bacteria enter the blood, it leads to activation of neutrophils, monocytes, interleukin and
cytokines. They rush the blood stream to attach the bacteria. Bacteria possess different
endotoxins. These toxins damage the endothelial cells walls and also activate the macrophages,
interleukins, cytokines, and neutrophils leading to systemic vasodilation. This systemic
vasodilation further causes hypotension and underperfusion of the tissues causing lactic
acidosis. Coagulopathy occurs as the endothelium of the blood vessels gets damaged by the
toxins. Eventually body organs begin to damage or organ failure ensues. Bacterial infection and
sepsis lead to decrease systemic vascular resistance and cardiac output increases to
compensate this.

Sepsis parameters include: decreased systemic vascular resistance, increased cardiac output
and decreased capillary pulmonary wedge pressure.

1. SIRS
IN this condition following important findings are present:
Fever > 38C or <36C
HR and pulse > 90 beats/min
Tachypnea (develops due to lactic acidosis and so to blow out excess CO2 it leads to
hyperventiliation) Respiratory Rate is greater than 20 whole PCO2 is less than 32 mmHg.
WBC > 12,000 cells/ml or <4000 cells/ml or 10% band (if person is immunocompromise).
There may therefore be leukocytosis or leukopenia.

2. Sepsis
It is defined as SIRS (fever and increased heart rate) plus infection source. It can be understood
by following:
Fever and neck rigidity it means person is suffering from meningitis
Fever and productive cough signify pneumonia
Fever and flank pain--- define to be pyelonephritis
Fever and redness give hint for cellulitis
Fever and right upper quadrant pain it means person is suffering from ascending cholangitis.

3. Severe Sepsis
Severe sepsis is explained as:
Lactate levels > 4mm/dl
Oliguria <0.5 ml/kg/hour
Change in mental status such as confusion or lethargic
Molted skin
Thrombocytopenic < 100,000 leading to coagulopathy
ARDS injury

Complications
Acute respiratory distress syndrome (ARDS)
Disseminated intravascular coagulation (DIC)
Acute tubular necrosis (ATN)
Multi organ dysfunction syndrome (MODS)
Death

Clinical symptoms and signs
Hypotension
Oliguria
Altered mental status
Warm skin
fever

Diagnosis
Sepsis is diagnosed on the basis of Clinical picture i.e. based on symptoms &
Blood cultures

Treatment
Intravenously broad spectrum antibiotics are given at maximum dose.
If there is abscess then surgical drainage is used.
IV fluids administration is must and important. Normal saline is usually given intravenously.
Vasopressors are also administered to support perfusion and heart. These include
dopamine, Dubutamine or norepinephrine. Dopamine is given to increase renal perfusion

Quiz

1. A previously healthy 33 year old man presents to emergency with septic shock secondary to
cellulitis of the arm. He is slightly confused and his peripheries are cold. His heart rate is
125/min, BP 70/30 mmHg, and respiratory rate 25/min. What should be the most suitable
first line management in this case?
A. Steroids
B. Analgesics
C. Broad spectrum antibiotics
D. Antipyretics

The correct answer is: C

In this case the most suitable first line management is the broad spectrum antibiotics. The
rationale behind is that this patient has a previous history of cellulitis and due to which he has
now developed a shock. So to combat the infection, antibiotics are essential.

Steroids are usually not given or given only when there is a presence of inflammation.

Analgesics are not used here as patient has not complained of any kind of pain.

Antipyretics are given to treat fever but these are not the first line of management of septic
shock.

2. A febrile patient was diagnosed with E. coli sepsis. Shorty, he develops septic shock. Which
of the given option is responsible for this reaction?
A. Bacterial surface antigens causing stimulation of a humoral immune response
B. A cell-mediated immune response to E.coli
C. Cytokines secretion by monocytes as a result of stimulation by endotoxin
D. Granule contents released by basophils and mast cells
E. Bacterial superantigen toxin causing activation of TH cells

The correct answer is: C.


The correct option is C. cytokines are responsible for the occurrence of septic shock. Cytokines
are secreted by the monocytes owing to the stimulation by the bacterial endotoxins. Cytokines
are basically immunomodulating agents.

Bacterial surface antigens causing stimulation of a humoral immune response does not lead to
this picture but the toxin produced by E.coli is responsible for septic shock.

A cell-mediated immune response also takes place to E.coli but again it does not give rise to
septic shock. Cell mediated immunity in fact offers protection against majority of intracellular
bacterial pathogens.

Granule contents released by basophils and mast cells do not lead to septic shock. Instead
anaphylactic shock may occur if there is severe allergic reaction taken place.

Bacterial superantigen toxin causing activation of TH cells leads to release of cytokine but can
produce the picture of Toxic Shock Syndrome not the septic shock.

3. A patient with septic shock develops hypotension. What is the cause behind hypotension?
A. Increased capillary permeability and massive vasodilation
B. Decreased systemic vascular resistance and vasoconstriction
C. Decreased capillary permeability
D. Massive vasoconstriction

The correct answer is: A

Increased capillary permeability and massive vasodilation are both held responsible for
producing hypotension in septic shock. The increase in capillary permeability as well as
vasdilation occurs as a result of inflammatory mediators affecting the capillaries. The systemic
immune response to microbial infection brings about venous blood pooling and arterial
vasodilation.

Vasoconstriction does not occur here instead vasodilation takes place. The peripheral vascular
resistance is however decreased here.

Capillary permeability is not decreased but it is increased due to systemic immune response to
endotoxins.

Massive vasoconstriction is not possible so this option is not correct.


4. A patient presents with confusion. He has a history of fever and vomiting for 5 days. On
examination his skin is warm, neck is rigid, pulse is 120/minute and BP is 90/70mmHg. What
is the most probable diagnosis?
A. Meningitis
B. Sepsis and meningitis
C. Sepsis
D. Neurogenic shock

The correct answer is: B

The clinical picture goes with the diagnosis of sepsis and meningitis. Fever, vomiting and neck
rigidity show increased intracranial pressure. This is most likely due to meningitis while
confusion, decreased BP, hot flushed skin and increased heart rate show the occurrence of
sepsis (i.e. fever and source of infection).

Although meningitis is present but there is an element of sepsis as well so alone meningitis is
not a correct option.

Likewise, sepsis alone is not correct since there is a clear picture of meningitis.

Neurogenic shock may present with decreased BP, but history of fever is unlikely. However,
there is a history of trauma to back or spinal cord. Additionally, skin of the patient is usually
cold here due to hypotension.

5. A patient presents with molted skin, oliguria, hypotension, confusion, elevated lactate and
purpura all over the skin. What would be complication of sepsis causing these problems?
A. Acute respiratory distress syndrome (ARDS)
B. Metabolic acidosis
C. Systemic inflammatory response syndrome (SIRS)
D. Multi organ dysfunction syndrome (MODS)

The correct answer is: D

Molted skin, oliguria, hypotension, confusion, elevated lactate and purpura all over the skin
demonstrate the occurrence of multi organ dysfunction syndrome (MODS). It is one of the most
common complication that affect different organs of the body.

If an acute respiratory distress syndrome (ARDS) occurs, then patient complains of shortness of
breath or difficult breath.
Metabolic acidosis does not produce these symptoms. It may present with headache, pain in
chest, palpitations, weakness, etc.

Systemic inflammatory response syndrome (SIRS) is an inflammatory condition affecting the


whole body, and is mainly a response of the bodys immune system to infection. Temperature is
either high or low; there is tachypnea and increased heart rate. Oliguria, hypotension,
confusion, elevated lactate and purpura are not present here.


Future teaching physicians Lectures LLC
Medicine made simple






Title: Stable Angina

Objectives for learning: Learning pathophysiological and clinical features of stable
angina; understanding the importance of correct diagnosis and treatment.

Definitions: Stable angina is an imbalance between blood flow through the
coronary arteries and oxygen demand.

Causes/ Risk factors:
Acronym: FLASH MD
Family history of coronary disease
Low HDL (<40)
Age (men>45, women>55)
Smoking
Hypertension
Male gender
Diabetes

Pathophysiology:
Due to atherosclerotic plaques that occlude the lumen of coronary arteries, there is
a higher demand for oxygen than it could be delivered through the occluded
arteries. Decreased perfusion to the heart muscle causes ischemia of the
myocardium presented with chest pain, especially during the exercise.

Clinical symptoms and signs
Chest pain
Exertion
Symptoms can last for 10 15 min, but usually 1-5 min.
Symptoms are relieved by rest and sublingual nitroglycerin
Diagnosis
EKG (normal findings)
Cardiac enzyme (no elevation)
Stress test
o Exercise (treadmill) until the maximum heart rate is reached (220
age)
Chest pain
Hypotension
ST-depression
Arrhythmia
o Stress echo
Cardiac catheterization if stress test is positive.
Pharmacologic stress test (for patients that are not able to perform exercise)
o IV adenosine (causes coronary vasodilatation)
o Dipyridamol (causes coronary vasodilatation)
o Dobutamine (increases myocardial oxygen demand; increases heart
rate, contractility, and blood pressure)

Complications
Progression of stable angina leads to acute coronary syndrome (unstable angina,
non-STEMI, and STEMI)

Treatment
Risk factor modification
o Diet (<7% saturated fat; <200 mg/day cholesterol)
o Exercise
o Lose weight
o Strict blood glucose control
o Statins
o Blood pressure control
o Stop smoking
Medication
o Aspirin
o Beta-blockers
o Nitrates
Revascularization
o PTCA (percutaneous transluminal coronary angioplasty) stent
implantation
o Coronary bypass
Title: Tricuspid Regurgitation

Objectives for learning: Definition, Causes/ Risk factors, Clinical symptoms and signs and v

Definition:
It is the inability of three valves (tricuspid valves) to close completely during systole.

Causes/ Risk factors:


The most common cause is right ventricular dilation. Other causes include:
Left ventricular failure
Right ventricular infraction
Cor pulomonale (pulmonary hypertension)
Staph aureus infection
Ebstein anomaly (congenital heart disease)
Carcinoid syndrome

Pathophysiology:

Clinical symptoms and signs:

Ascites: Jugular Venous Distention, heptomegaly


Pulsatile liver
Blowing holo-systolic murmur: When patient is asked to deep breath, the murmur is heard
louder.
Atrial fibrillation

Diagnosis:

Treatment:
Treat the underlying cause
Surgery involve valve repairmen or valvuloplasty

Quiz

1. A patient came with leg edema and swollen abdomen. On examination jugular vein was
found to be distended, ascites were present and liver was enlarged and pulsatile. What is the
most likely diagnosis?
A. Aortic stenosis
B. Pulmonary stenosis
C. Tricuspid regurgitation
D. Mitral regurgitation

The most likely diagnosis based on the clinical symptoms and signs is the tricuspid regurgitation.
In this condition tricuspid valve fails to close during systole, causing the blood to passes with
each heart beat from the right ventricle to the right atrium, i.e. in the direction opposite to that of
the normal one. Symptoms of right-sided heart failure often develop. Jugular vein is usually
distended. The important finding of tricuspid regurgitation is the pulsatile liver. It is not present
in the other conditions mentioned in the options.

Aortic stenosis is the condition of narrowing of the aortic valve. It may present with pedal,
exertional dyspnea, paroxysmal nocturnal dyspnea, but liver is not pulsatile here.

Pulmonary stenosis is the condition of obstruction of blood flow from the right ventricle to
the pulmonary artery. Due to this right ventricular hypertrophy develops. There may be
complaint of chest pain, palpitations, dysnpea but liver is not pulsatile.

Mitral regurgitation is characterized by the symptoms of decompensated congestive heart failure.


Liver here is not pulsatile but normal.

2. Which of the given option is associated with pansystolic murmur?

A. Aortic stenosis ejection systolic


B. Pulmonary stenosis
C. Tricuspid regurgitation
D. Pulmonary regurgitation decrescendo diastolic murmur

Tricuspid regurgitation is associated with pansystolic murmur. This type of murmur is of low
frequency and shows tendency to increase with inspiration.

Aortic stenosis is associated with ejection systolic murmur.

The murmur of pulmonary stenosis is systolic murmur.

Pulmonary regurgitation is associated with decrescendo diastolic murmur.

3. Identify the condition in which pulsatile liver is found?

A. Tricuspid regurgitation
B. Mitral regurgitation
C. Pulmonary hypertension
D. Mitral stenosis

Liver becomes pulsatile in case of tricuspid regurgitation. This condition is responsible for liver
dysfunction.

Mitral regurgitation, pulmonary hypertension and mitral stenosis do not lead to the liver to
become pulsatile. Liver usually remains normal.
4. What is the surgical management of tricuspid regurgitation?

A. Removal of valves
B. There is no surgical treatment of tricuspid regurgitation
C. Only repair of the valve
D. Valve repair (valvuloplasty) or replacement

The surgical management of tricuspid regurgitation is Valve repair (valvuloplasty) or


replacement. The valves are repaired surgically to make them functional.

Valves cannot be removed but can be replaced for the normal functioning of the heart.

The surgical treatment of tricuspid regurgitation is present and is successful.

Valve can be repaired or can be replaced depending upon the condition.

5. A 9-year-old boy is found to have tricuspid regurgitation. What would be most common
cause behind this?
A. Atrial septal defect
B. Ventricular septal defect
C. Ebstein anomaly
D. Carcinoid syndrome

The most common cause in the 9-year-boy is Ebstein anomaly. Ebstein anomaly is a basically a
congenital cardiac defect characterized by the displacement of septal leaflet of the tricuspid
valve towards the right ventricular apex.

Atrial septal defect is not presented with the tricuspid regurgitation. It is basically a defect
between the two atria, right and left, causing the oxygenated blood to mix with the deoxygenated
blood.

Ventricular septal defect as the name implies is the congenital anomaly characterized by the
presence of defect ventricular septum. This condition is not presented with tricuspid
regurgitation. If defect is small, it remains asymptomatic or disease usually becomes apparent a
few weeks after birth.

Carcinoid syndrome is the complex of different symptoms such as diarrhea, dysnpea, flushing,
palpitations etc. this syndrome is due to carcinoid tumor. Although tricuspid regurgitation occurs
in carcinoid syndrome but in this patient, no such history is there regarding the symptoms of
carcinoid syndrome. Also, this syndrome is more common in adults. Thus, it is not considered a
correct option here.

Title: Understanding The Basics Of Shock

Objectives for learning: Definition, Types, Complications, Symptoms, Diagnosis and Treatment
of shock.

Definition:
Shock is the under perfusion of the tissues. Blood flow is important for the normal functioning
of the brain, heart, kidney, liver, muscles and different other body tissues. Blood delivers
oxygen, nutrients and glucose needed for metabolism. It is a serious medical emergency and
therefore requires an immediate medical intervention otherwise it will prove fatal if irreversible
organ damage occurs.

Parameters going to be affected


Cardiac output
Systemic resistance
Pulmonary capillary wedge pressure

Causes/ Risk factors:

Pathophysiology

Clinical symptoms and signs


Hypotension (low BP)
Oliguria
Tachycardia
Altered mental status

Complications of Shocks
Lactic acidosis (anaerobic respiration)
Oliguria (decreased urine output)
CNS dysfunction (altered mental status)

Types of Shock
Neurogenic shock
Cardiogenic shock
Anaphylactic shock
Septic shock
Hypvolemic shock
Diagnosis/ Approach to shock
History: If there is history of fever and low BP, it shows infection (it may be a septic shock).
If there is history of trauma, GI bleed, vomiting, or diarrhea, it means volume depletion has
occurs so hypovolemic shock is possible.
If there is complaint of chest pain and low BP and cardiac output is decreased, a person may be
having acute myocardial infarction (MI), then it will be a cardiogenic shock.
In case of neurologic shock there will be neurologic deficit.

IV infusions is important and given with 2 large bored IV lines. A fluid of about 500 to 1000 ml
should be given for organ perfusion.

Labs include: blood sampling for CBC, PT/APTT, RFTs (BUN/Creatinine) and BMP
EKG: To check the presence of ST elevation, depression or other possible findings
Chest X-ray: It will help in identifying the chronic heart failure (CHF), cardiomegaly, tension
pneumothorax
Pulse oximetry: It should be 100%

Vasopressors are given such as dopamine, dubutamine and norepinephrine.

Treatment
The basic and most important early treatment of shock involves:
ABC
i.e.
Maintain airways
Breathing
Circulation

Other treatment varies depending on the type of shock and its causes.


Quiz
1. A 14-year-patient presents with fever, confusion and weakness. On examination he is pale
with weak thread pulse. His temperature is 102F. His attendant tells that patient has been
suffering from flank pain for 4 days before. His BP is 80/60mmHg. What is the most likely
diagnosis?
A. Acute Kidney Injury
B. Shock
C. Adrenal Crisis
D. Toxic Shock Syndrome

The correct answer is B.

This patient is a typical case of shock, most probably the septic shock. This is because there is a
history of flank pain and fever. His temperature is also raised. Low BP and altered mental status
indicate that he has developed a shock.

Acute kidney injury is not presented with low BP. It is presented mostly with high BP and there
is generalized edema. Patient complains of headache and vomiting.

Adrenal Crisis is characterized by severe adrenal insufficiency due to inadequate levels of


cortisol in the body. There is although symptoms of confusion, fever, and low BP but there is a
wide range of metabolic abnormalities such as hyperkalemia, hypercalcemia, hypoglycemia and
hyponatremia.

Toxic shock syndrome (TSS) is a acute life-threatening condition mediated by bacterial toxin
usually by either Staphylococcus aureus or Streptococcus. It is characterized by rash, high fever,
desquamation hypotension, and multi-organ failure. It is presented with severe myalgia,
headache, diarrhea, vomiting, and non-focal neurologic deficit.

2. In a patient with reduced cardiac output and low BP with the diagnosis of shock, what
would be the best treatment option to boost cardiac output?
A. IV Fluids
B. Beta Blockers
C. Antibiotics
D. Vasopressors

The correct answer is D.


The best treatment option to enhance the cardiac output and low BP in patient with shock is
the vasopressors. These drugs work by increasing the BP, contractility of the heart as well as the
renal perfusion.

IV fluids are although must here but sometimes they fail to increase the BP and cardiac output.

Beta Blockers have no role in increasing the heart rate or blood pressure; in fact they reduce
blood pressure and tachycardia.

Antibiotics are usually prescribed when there is any evidence of infection. They have no role in
increasing the cardiac output and BP.

3. Which of the given feature helps to differentiate the hypovolemic shock from septic shock?
A. Blood pressure
B. Cardiac Output
C. Temperature
D. Heart rate

The correct answer is C.

Body temperature is the differentiating feature in case of the hypovolemic shock and septic
shock. In hypvolemic shock temperature is reduced so patients skin is cold while in septic shock
due to infection temperature is raised so skin is warm.

Blood pressure and cardiac output both are reduced in both hypvolemic and septic shock. So
these parameters cannot help in differentiating.

Likewise, heart rate is also increased in both shock therefore, it wont help in confirming
whether a patient is suffering from hypovolemic and septic shock.

4. A patient presents with the progressive stage of shock. What will happen with the
metabolism of patients if the shock is not treated and hypoxia of the tissue occurs?
A. Compensatory mechanisms
B. Lactic acidosis
C. Arteriolar constriction due to vasomotor reflex
D. Metabolism will be unaffected

The correct answer is B.

A patient with the progressive stage of shock, if not treated and developed hypoxia of the
tissue, most likely enters into anaerobic metabolism and therefore develops lactic acidosis.
Compensatory mechanisms are already working well before the person enters into progressive
shock, so this option is not correct.

Arteriolar constriction due to vasomotor reflex is again already working here to compensate.

Metabolism will be affected here when oxygen delivery and nutrients are not sufficient so to
compensate aerobic metabolism shifts to anaerobic.

5. A patient presents with confusion after severe bleeding in the vomiting. He has developed
tachycardia while his pulse is weak. Which kind of shock is associated with low levels of
blood?

A. Hypovolemic shock
B. Anaphylactic shock
C. Cardiogenic shock
D. Septic shock

The correct answer is A

Hypovolemic shock is associated with low levels of blood and is characterized by bleeding or
hemorrhage.

Anaphylactic shock is associated with allergic reaction to drug, chemical, etc and is the serious
sort of allergic reaction.

Cardiogenic shock is characterized by different conditions affecting the heart causing the heart
difficult to pump the blood as the body requires.

Septic shock occurs after the infection/bacteria enter into the blood stream and is an extreme
response by the immune system.

Potrebbero piacerti anche